\documentclass[10pt,a4paper,oneside,ngerman,numbers=noenddot]{scrartcl} \usepackage[T1]{fontenc} \usepackage[utf8]{inputenc} \usepackage[ngerman]{babel} \usepackage{amsmath} \usepackage{amsfonts} \usepackage{amssymb} \usepackage{paralist} \usepackage{gauss} \usepackage{pgfplots} \usepackage[locale=DE,exponent-product=\cdot,detect-all]{siunitx} \usepackage{tikz} \usetikzlibrary{matrix,fadings,calc,positioning,decorations.pathreplacing,arrows,decorations.markings} \usepackage{polynom} \polyset{style=C, div=:,vars=x} \pgfplotsset{compat=1.8} \pagenumbering{arabic} % ensures that paragraphs are separated by empty lines \parskip 12pt plus 1pt minus 1pt \parindent 0pt % define how the sections are rendered \def\thesection{\arabic{section})} \def\thesubsection{\alph{subsection})} \def\thesubsubsection{(\roman{subsubsection})} % some matrix magic \makeatletter \renewcommand*\env@matrix[1][*\c@MaxMatrixCols c]{% \hskip -\arraycolsep \let\@ifnextchar\new@ifnextchar \array{#1}} \makeatother \begin{document} \author{Jan Branitz (6326955), Jim Martens (6420323),\\ Stephan Niendorf (6242417)} \title{Hausaufgaben zum 4. November} \maketitle \section{} %1 \subsection{} %a \textbf{Aufgabe:} Lösen Sie das folgende LP-Problem mit dem Simplexverfahren: \begin{alignat*}{3} \text{maximiere}\; & x_{1} &+& 2x_{2} && \\ \multicolumn{6}{l}{\text{unter den Nebenbedingungen}} && \\ \;-& 4x_{1} &+& x_{2} &\leq & 1 \\ \;-& x_{1} &+& x_{2} &\leq & 2 \\ \;& \frac{1}{2}x_{1} &-& x_{2} &\leq & 1 \\ \multicolumn{4}{r}{$x_{1}, x_{2}$} \,&\geq &\, 0 \end{alignat*} \textbf{Lösung.} \underline{Starttableau}: \begin{alignat*}{4} x_{3} \,&=&\, 1 \,&+&\, 4x_{1} \,&-&\, x_{2} \\ x_{4} \,&=&\, 2 \,&+&\, x_{1} \,&-&\, x_{2} \\ x_{5} \,&=&\, 1 \,&-&\, \frac{1}{2}x_{1} \,&+&\, x_{2} \\ \cline{1 - 9} z &=& && x_{1} \,&+&\, 2x_{2} \end{alignat*} \underline{1. Iteration}: Eingangsvariable: $x_{2}$\\ Ausgangsvariable: $x_{3}$ Es folgt \begin{alignat*}{2} x_{2} \,&=&&\, 1 + 4x_{1} - x_{3} \\ x_{4} \,&=&&\, 2 + x_{1} - \left(1 + 4x_{1} - x_{3}\right) \\ &=&&\, 2 + x_{1} - 1 - 4x_{1} + x_{3} \\ &=&&\, 1 - 3x_{1} + x_{3} \\ x_{5} \,&=&&\, 1 - \frac{1}{2}x_{1} + \left(1 + 4x_{1} - x_{3}\right) \\ &=&&\, 1 - \frac{1}{2}x_{1} + 1 + 4x_{1} - x_{3} \\ &=&&\, 2 + \frac{7}{2}x_{1} - x_{3} \\ z \,&=&&\, x_{1} + 2\left(1 + 4x_{1} - x_{3}\right) \\ &=&&\, x_{1} + 2 + 8x_{1} - 2x_{3} \\ &=&&\, 2 + 9x_{1} - 2x_{3} \end{alignat*} \underline{Ergebnis der 1. Iteration}: \begin{alignat*}{4} x_{2} \,&=&\, 1 \,&+&\, 4x_{1} \,&-&\, x_{3} \\ x_{4} \,&=&\, 1 \,&-&\, 3x_{1} \,&+&\, x_{3} \\ x_{5} \,&=&\, 2 \,&+&\, \frac{7}{2}x_{1} \,&-&\, x_{3} \\ \cline{1 - 9} z &=& 2 \,&+&\, 9x_{1} &-& 2x_{3} \end{alignat*} \underline{2. Iteration}: Eingangsvariable: $x_{1}$ \\ Ausgangsvariable: $x_{4}$ Es folgt \begin{alignat*}{2} 3x_{1} &=&& 1 + x_{3} - x_{4} \\ x_{1} &=&& \frac{1}{3} + \frac{1}{3}x_{3} - \frac{1}{3}x_{4} \\ x_{2} &=&& 1 + 4\left(\frac{1}{3} + \frac{1}{3}x_{3} - \frac{1}{3}x_{4}\right) - x_{3} \\ &=&& 1 + \frac{4}{3} + \frac{4}{3}x_{3} - \frac{4}{3}x_{4} \\ &=&& \frac{7}{3} + \frac{4}{3}x_{3} - \frac{4}{3}x_{4} \\ x_{5} &=&& 3 + \frac{7}{2}\left(\frac{1}{3} + \frac{1}{3}x_{3} - \frac{1}{3}x_{4}\right) - x_{3} \\ &=&& 3 + \frac{7}{6} + \frac{7}{6}x_{3} - \frac{7}{6}x_{4} - x_{3} \\ &=&& \frac{13}{6} + \frac{1}{6}x_{3} - \frac{7}{6}x_{4} \\ z &=&& 2 + 9\left(\frac{1}{3} + \frac{1}{3}x_{3} - \frac{1}{3}x_{4}\right) - 2x_{3} \\ &=&& 2 + 3 + 3x_{3} - 3x_{4} - 2x_{3} \\ &=&& 5 + x_{3} - 3x_{4} \end{alignat*} \underline{Ergebnis der 2. Iteration}: \begin{alignat*}{4} x_{1} \,&=&\, \frac{1}{3} \,&+&\, \frac{1}{3}x_{3} \,&-&\, \frac{1}{3}x_{4} \\ x_{2} \,&=&\, \frac{7}{3} \,&+&\, \frac{4}{3}x_{3} \,&-&\, \frac{4}{3}x_{4} \\ x_{5} \,&=&\, \frac{13}{6} \,&+&\, \frac{1}{6}x_{3} \,&-&\, \frac{7}{6}x_{4} \\ \cline{1 - 7} z &=& 5 \,&+&\, x_{3} \,&-&\, 3x_{4} \end{alignat*} \underline{3. Iteration}: Eingangsvariable: $x_{3}$ \\ Ausgangsvariable: keine vorhanden Es gibt keine optimale Lösung, da es keine Basisvariable gibt, die $x_{3}$ beschränkt. Damit ist dieses Problem unbeschränkt. \underline{Startlösung ("`zulässige Basislösung am Anfang"')}: \[ x_{1} = 0, x_{2} = 0, x_{3} = 1, x_{4} = 2, x_{5} = 1 \text{ mit } z = 0 \] \underline{Zulässige Basislösung nach der 1. Iteration}: \[ x_{1} = 0, x_{2} = 1, x_{3} = 0, x_{4} = 1, x_{5} = 2 \text{ mit } z = 2 \] \underline{Zulässige Basislösung nach der 2. Iteration}: \[ x_{1} = \frac{1}{3}, x_{2} = \frac{7}{3}, x_{3} = 0, x_{4} = 0, x_{5} = \frac{13}{6} \text{ mit } z = 5 \] Ermittlung einer Halbgeraden des $\mathbb{R}^{2}$: \begin{alignat*}{2} x_{3} &=& t \\ x_{4} &=& 0 \\ x_{1} &=& \frac{1}{3} + \frac{1}{3}t \\ x_{2} &=& \frac{7}{3} + \frac{4}{3}t \\ x_{5} &=& \frac{13}{6} + \frac{1}{6}t \\ z &=& 5 + t \\ t &\geq & 0 \end{alignat*} Daraus ergibt sich in Parameterform: \begin{alignat*}{2} (x_{1}, x_{2}) &=& \left(\frac{1}{3} + \frac{1}{3}t, \frac{7}{3} + \frac{4}{3}t\right) \\ &=& \left(\frac{1}{3}, \frac{7}{3}\right) + t\left(\frac{1}{3}, \frac{4}{3}\right) \end{alignat*} %\begin{alignat*}{2} % (x_{3}, x_{2}) &=& \left(t, \frac{7}{3} + \frac{4}{3}t\right) \\ % &=& \left(0, \frac{7}{3}\right) + t\left(1, \frac{4}{3}\right) %\end{alignat*} Da in diesem Fall $x_{1}$ eine Basisvariable ist und somit nicht gleich $t$ ist, stellt $t$ eine beliebige positive Konstante dar. Daher verändert sich auch die Gerade je nach Wahl von $t$. Deswegen ist es nicht möglich genau eine Halbgerade zu finden, auf der die Zielfunktion beliebig große Werte annimmt. \subsection{} %b Durch Umstellen der Nebenbedingungen des Problems aus a nach $x_{2}$ ergibt sich: \begin{alignat*}{3} x_{2} &\leq & 4x_{1} &+& 1 \\ x_{2} &\leq & x_{1} &+& 2 \\ x_{2} &\geq & \frac{1}{2}x_{1} &-& 1 \end{alignat*} Daraus lässt sich die Fläche aller gültigen Werte zeichnen. \begin{tikzpicture}[>=stealth] \begin{axis}[ ymin=0,ymax=7, x=1cm, y=1cm, axis x line=middle, axis y line=middle, axis line style=->, xlabel={$x_{1}$}, ylabel={$x_{2}$}, xmin=0,xmax=7 ] \addplot[no marks, black, -] expression[domain=0:6,samples=100]{4*x + 1} node[pos=0.65,anchor=north]{}; \addplot[no marks, black, -] expression[domain=0:6,samples=100]{1*x + 2} node[pos=0.65,anchor=north]{}; \addplot[no marks, black, -] expression[domain=0:6,samples=100]{0.5*x - 1} node[pos=0.65,anchor=north]{}; %\addplot[no marks, black, -] expression[domain=0:6,samples=100]{1.333333333333333*x + 2.33333333333333333} node[pos=0.65,anchor=north]{}; %\node at (axis cs: 2.5,4.5) {(2.25,3.75)}; %\node at (axis cs: 6,2) {z}; %\draw[>=stealth] (axis cs:1,0) -- (axis cs:1,-6) node [pos=0.65,anchor=north]{}; \end{axis} \end{tikzpicture}\\ \section{} %2 \subsection{} %a \textbf{Aufgabe:} Lösen Sie das folgende LP-Hilfsproblem mit dem Simplexverfahren: \begin{alignat*}{5} \text{maximiere}\; -& x_{0} && && && && \\ \multicolumn{10}{l}{\text{unter den Nebenbedingungen}} && \\ \; & &-& x_{1} &-& x_{2} &-& x_{0} &\leq & -4 \\ \; &&& x_{1} &+& 2x_{2} &-& x_{0} &\leq & 2 \\ \; &&-&x_{1} &+& x_{2} &-& x_{0} &\leq & 1 \\ \multicolumn{8}{r}{$x_{0}, x_{1}, x_{2}$} \,&\geq &\, 0 \end{alignat*} \textbf{Lösung.} \underline{Starttableau}: \begin{alignat*}{5} x_{3} \,&=&\, -4 \,&-&\, x_{1} \,&+&\, x_{2} \,&+&\, x_{0} \\ x_{4} \,&=&\, 2 \,&-&\, x_{1} \,&-&\, 2x_{2} \,&+&\, x_{0} \\ x_{5} \,&=&\, 1 \,&+&\, x_{1} \,&-&\, x_{2} \,&+&\, x_{0} \\ \cline{1 - 9} w &=& && && \,&-&\, x_{0} \end{alignat*} Umwandeln in ein zulässiges Tableau: Eingangsvariable: $x_{0}$\\ Ausgangsvariable: $x_{3}$ Es folgt \begin{alignat*}{2} -x_{0} \,&=&&\, -4 - x_{1} + x_{2} - x_{3} \\ x_{0} \,&=&&\, 4 + x_{1} - x_{2} + x_{3} \\ x_{4} \,&=&&\, 2 - x_{1} - 2x_{2} + \left(4 + x_{1} - x_{2} + x_{3}\right) \\ &=&&\, 2 - x_{1} - 2x_{2} + 4 + x_{1} - x_{2} + x_{3} \\ &=&&\, 6 - 3x_{2} + x_{3} \\ x_{5} \,&=&&\, 1 + x_{1} - x_{2} + \left(4 + x_{1} - x_{2} + x_{3}\right) \\ &=&&\, 1 + x_{1} - x_{2} + 4 + x_{1} - x_{2} + x_{3} \\ &=&&\, 5 + 2x_{1} - 2x_{2} + x_{3} \\ w \,&=&&\, -\left(4 + x_{1} - x_{2} + x_{3}\right) \\ &=&&\, -4 - x_{1} + x_{2} - x_{3} \\ \end{alignat*} \underline{Ergebnis des Umwandelns}: \begin{alignat*}{5} x_{0} \,&=&\, 4 \,&+&\, x_{1} \,&-&\, x_{2} \,&+&\, x_{3} \\ x_{4} \,&=&\, 6 \,&& &-&\, 3x_{2} \,&+&\, x_{3} \\ x_{5} \,&=&\, 5 \,&+&\, 2x_{1} \,&-&\, 2x_{2} \,&+&\, x_{3} \\ \cline{1 - 9} w &=& -2 \,&-&\, x_{1} \,&+&\, x_{2} \,&-&\, x_{3} \end{alignat*} \underline{1. Iteration}: Eingangsvariable: $x_{2}$ \\ Ausgangsvariable: $x_{4}$ Es folgt \begin{alignat*}{2} 3x_{2} &=&& 6 + x_{3} - x_{4} \\ x_{2} &=&& 2 + \frac{1}{3}x_{3} - \frac{1}{3}x_{4} \\ x_{0} &=&& 4 + x_{1} - \left(2 + \frac{1}{3}x_{3} - \frac{1}{3}x_{4}\right) + x_{3} \\ &=&& 4 + x_{1} - 2 - \frac{1}{3}x_{3} + \frac{1}{3}x_{4} + x_{3}\\ &=&& 2 + x_{1} + \frac{2}{3}x_{3} + \frac{1}{3}x_{4} \\ x_{5} &=&& 5 + 2x_{1} - 2\left(2 + \frac{1}{3}x_{3} - \frac{1}{3}x_{4}\right) + x_{3} \\ &=&& 5 + 2x_{1} - 4 - \frac{2}{3}x_{3} + \frac{2}{3}x_{4} + x_{3} \\ &=&& 1 + 2x_{1} + \frac{1}{3}x_{3} + \frac{2}{3}x_{4} \\ w &=&& -2 - x_{1} + \left(2 + \frac{1}{3}x_{3} - \frac{1}{3}x_{4}\right) - x_{3} \\ &=&& -2 - x_{1} + 2 + \frac{1}{3}x_{3} - \frac{1}{3}x_{4} + x_{3} \\ &=&& 0 - x_{1} + \frac{4}{3}x_{3} - \frac{1}{3}x_{4} \end{alignat*} \underline{Ergebnis der 1. Iteration}: \begin{alignat*}{5} x_{2} \,&=&\, 2 \,&& &+&\, \frac{1}{3}x_{3} \,&-&\, \frac{1}{3}x_{4} \\ x_{0} \,&=&\, 2 \,&+&\, x_{1} \,&+&\, \frac{2}{3}x_{3} \,&+&\, \frac{1}{3}x_{4} \\ x_{5} \,&=&\, 1 \,&+&\, 2x_{1} \,&+&\, \frac{1}{3}x_{3} \,&+&\, \frac{2}{3}x_{4} \\ \cline{1 - 9} w &=& 0 \,&-&\, x_{1} \,&+&\, \frac{4}{3}x_{3} \,&-&\, \frac{1}{3}x_{4} \end{alignat*} Da das Hilfsproblem keine optimale Lösung besitzt, besitzt das ursprüngliche Problem keine zulässige Lösung und ist damit unlösbar. \subsection{} %b \textbf{Aufgabe:} Lösen Sie das folgende LP-Hilfsproblem mit dem Simplexverfahren: \begin{alignat*}{5} \text{maximiere}\; -& x_{0} && && && && \\ \multicolumn{10}{l}{\text{unter den Nebenbedingungen}} && \\ \; & && x_{1} &-& x_{2} &-& x_{0} &\leq & 8 \\ \; & &-& x_{1} &-& x_{2} &-& x_{0} &\leq & -3 \\ \; & &-& x_{1} &+& 4x_{2} &-& x_{0} &\leq & 2 \\ \multicolumn{8}{r}{$x_{0}, x_{1}, x_{2}$} \,&\geq &\, 0 \end{alignat*} \textbf{Lösung.} \underline{Starttableau}: \begin{alignat*}{5} x_{3} \,&=&\, 9 \,&-&\, x_{1} \,&+&\, x_{2} \,&+&\, x_{0} \\ x_{4} \,&=&\, -3 \,&+&\, x_{1} \,&+&\, x_{2} \,&+&\, x_{0} \\ x_{5} \,&=&\, 2 \,&+&\, x_{1} \,&-&\, 4x_{2} \,&+&\, x_{0} \\ \cline{1 - 9} w &=& && && \,&-&\, x_{0} \end{alignat*} Umwandeln in ein zulässiges Tableau: Eingangsvariable: $x_{0}$\\ Ausgangsvariable: $x_{4}$ Es folgt \begin{alignat*}{2} -x_{0} \,&=&&\, -3 + x_{1} + x_{2} - x_{4} \\ x_{0} \,&=&&\, 3 - x_{1} - x_{2} + x_{4} \\ x_{3} \,&=&&\, 9 - x_{1} + x_{2} + \left(3 - x_{1} - x_{2} + x_{4}\right) \\ &=&&\, 9 - x_{1} + x_{2} + 3 - x_{1} - x_{2} + x_{4} \\ &=&&\, 12 - 2x_{1} + x_{4} \\ x_{5} \,&=&&\, 2 + x_{1} - 4x_{2} + \left(3 - x_{1} - x_{2} + x_{4}\right) \\ &=&&\, 2 + x_{1} - 4x_{2} + 3 - x_{1} - x_{2} + x_{4} \\ &=&&\, 5 - 5x_{2} + x_{4} \\ w \,&=&&\, -\left(3 - x_{1} - x_{2} + x_{4}\right) \\ &=&&\, -3 + x_{1} + x_{2} - x_{4} \\ \end{alignat*} \underline{Ergebnis des Umwandelns}: \begin{alignat*}{5} x_{0} \,&=&\, 3 \,&-&\, x_{1} \,&-&\, x_{2} \,&+&\, x_{4} \\ x_{3} \,&=&\, 12 \,&-&\, 2x_{1} \,&& &+&\, x_{4} \\ x_{5} \,&=&\, 5 \,&& &-&\, 5x_{2} \,&+&\, x_{4} \\ \cline{1 - 9} w &=& -3 \,&+&\, x_{1} \,&+&\, x_{2} \,&-&\, x_{4} \end{alignat*} \underline{1. Iteration}: Eingangsvariable: $x_{1}$ \\ Ausgangsvariable: $x_{0}$ Es folgt \begin{alignat*}{2} x_{1} &=&& 3 - x_{2} + x_{4} - x_{0} \\ x_{3} &=&& 12 - 2\left(3 - x_{2} + x_{4} - x_{0}\right) + x_{4} \\ &=&& 12 - 6 + 2x_{2} - 2x_{4} + 2x_{0} + x_{4} \\ &=&& 6 + 2x_{2} - x_{4} + 2x_{0} \\ x_{5} &=&& 5 - 5x_{2} + x_{4} \\ w &=&& -3 + \left(3 - x_{2} + x_{4} - x_{0}\right) + x_{2} - x_{4} \\ &=&& -3 + 3 - x_{2} + x_{4} - x_{0} + x_{2} - x_{4} \\ &=&& - x_{0} \end{alignat*} \underline{Ergebnis der 1. Iteration}: \begin{alignat*}{5} x_{1} \,&=&\, 3 \,&-&\, x_{2} \,&+&\, x_{4} \,&-&\, x_{0} \\ x_{3} \,&=&\, 6 \,&+&\, 2x_{2} \,&-&\, x_{4} \,&+&\, 2x_{0} \\ x_{5} \,&=&\, 5 \,&-&\, 5x_{2} \,&-&\, x_{4} \,&& \\ \cline{1 - 9} w &=& && && &-& x_{0} \end{alignat*} Das Tableau ist optimal. Als optimale Lösung des Hilfsproblem erhält man: \[ x_{0} =0, x_{1} = 3, x_{2} = 0 \] Als zulässige Lösung für das ursprüngliche Problem ergibt sich: \[ x_{1} = 3, x_{2} = 0 \] Die ursprüngliche Zielfunktion lautet $z = x_{1} + 3x_{2}$. Setzt man für $x_{1}$ die rechte Seite der Gleichung im obigen Tableau ein, erhält man: \[ z = 3 - x_{2} + x_{4} + 3x_{2} = 3 + 2x_{2} + x_{4} \] Daraus ergibt sich dieses Starttableau: \begin{alignat*}{4} x_{1} \,&=&\, 3 \,&-&\, x_{2} \,&+&\, x_{4} \\ x_{3} \,&=&\, 6 \,&+&\, 2x_{2} \,&-&\, x_{4} \\ x_{5} \,&=&\, 5 \,&-&\, 5x_{2} \,&-&\, x_{4} \\ \cline{1 - 7} z \,&=&\, 3 \,&+&\, 2x_{2} \,&+&\, x_{4} \end{alignat*} \end{document}